Some of the Missing Energy

Guest Post by Willis Eschenbach

The canonical equation describing the energy balance of the earth looks like this:

∆Q (energy added) = ∆U (energy lost) + ∆Ocean (energy moving in/out of the ocean)                                                          (Equation 1)

This has been modified in the current climate paradigm (e.g. see Kiehl) by substituting in the following:

∆U (energy lost) = [∆T (change in surface temperature) / S (climate sensitivity)]                                                           (Equation 2)

which gives us

∆Q (energy added) = [∆T (change in surface temperature) / S (climate sensitivity)] + ∆Ocean (energy moving in/out of the ocean) (Equation 3)

As I detailed in “Where Did I Put That Energy“, the problem is that the data doesn’t bear out the substitution. In the real world, ∆U is very different from ∆T/S. There’s a whole lot of energy missing. I think that some of it is here:

Figure 1. Tracing the path of a tiny bit of energy through a simplified climate system.

Why does this count as some of the missing energy?

Note that all of the energy goes into evaporating the molecule of water. As a result, there is no net change in the surface temperature. Since the definition of the climate sensitivity is ∆T/∆Q, and ∆T is zero, that means that for this entire transaction the climate sensitivity is zero.

It is important to remember that Equation 1 is still true, and this situation complies with Equation 1. The amount of energy entering the system equals the amount leaving plus ocean storage (zero in Fig. 1). However, it does not comply with equation 2 or 3.

This certainly qualifies as a possible mechanism for the missing energy. Response time is fast, and it can move huge amounts of energy from the surface to the condensation level and eventually to space. Also, it is outside the ambit of the the climate sensitivity calculation, since the climate sensitivity for this transaction is zero.

Is this all of the missing energy? Can’t be. The missing energy is moving in huge amounts in both directions, both into and out of the system. However, the mechanism above is one-way. It can remove energy from the system, but not add energy. I say the extra energy added in the other direction comes from clouds clearing out when the temperature drops. But that is another story for another post.

My conclusion? Climate sensitivity is not a constant, it is a function of temperature. Note for example that the warmer the water, the larger a percentage of the incoming energy takes the path illustrated in Fig. 1. The formation of the clouds and thunderstorms is also temperature dependent. All of which makes the climate sensitivity strongly temperature dependent.

As always, questions, corrections, and suggestions are more than welcome.

w.

PS – Please don’t say “but you left out the greenhouse gases”. Yes, I did, but in this case they have almost no effect. The transport of the heat to the upper troposphere takes place in the thunderstorm, so it is protected from thermal exchange with the troposphere. At the top of the troposphere, where it leaves the thunderstorm, there is little atmosphere of any kind. From there it is free to radiate to space with little interference.

And in any case, GHGs will only modify rather than rule the effect. Sure, we might end up with a bit of surface warming rather than zero as in the above analysis. But the essence of the transaction is that surface temperature is not directly coupled to radiation. This means that the substitution done to get Equation 3 is not correct.

PPS — In fact, the system above does more than have zero effect on the surface temperature. When the thunderstorm starts, albedo goes up, storm winds increase evaporation, cold wind and rain from aloft chill the surface, and other cooling mechanisms kick into gear. As a result, the surface ends up cooler than when the thunderstorm started, giving negative climate sensitivity. But that is another story for another post as well.

0 0 votes
Article Rating
155 Comments
Oldest
Newest Most Voted
Inline Feedbacks
View all comments
December 23, 2010 8:41 pm

Lots of issues with all eqs. First off: the energy balance should look like
Energy Balance (+ or -) = [1] Energy captured from the Sun – [2] Energy emitted to space – [3] energy captured by earth (land, ocean and air) +[4] energy emitted from Earth (core heat still being depleted from Earth’s birth and the gravity forces adding energy to our molten core as we rotate the solar system and the solar system rotates the galaxy).
The balance is of where we stand (gaining or losing energy) is determined by a very complex system of systems. The simpleton and partial equations we have seen are embarrassing. For example, how much energy is captured and held by land, sea and air? There are two very large and very dynamic energy sources (external energy from the Sun and internal energy from our molten core). There are various sinks and one path for dissipation (to space). I have yet to see defensible models of all 4 factors and their error bars. Until then – no one knows.

Carl Chapman
December 23, 2010 8:46 pm

The models assume positive feedback will multiply the CO2 caused rise by a factor of 3. Assuming positive feedback in a relatively stable system is ridiculous. My guess a couple of years ago was that negative feedback would reduce the CO2 caused rise to 1/3 of what it would be without feedbacks. That would make the IPCC and models predictions out by a factor of 9.
Could you plug my guess into your equation please. Could you use a sensitivity such that the extra incoming energy that would normally need a 1 degree rise in T to restore balance without feedback, instead only needs a 1/3 degree rise in T.

Brian H
December 23, 2010 8:52 pm

Yum. Loverly.
I’ve always favoured a water-cycle based approach put together by a software guy called Robert Clemenzi: http://qs.mc-computing.com//Global_Warming/EPA_Comments/TheGreenhouseEffect.doc
He postulates and diagrams a “heat pipe – like” effect, with the thunderstorm piercing the lower troposphere and sending energy to the escape hatch in and above the tropopause.

December 23, 2010 9:03 pm

Willis:
This is “off topic”, yet ON TOPIC..
http://ntrs.nasa.gov/archive/nasa/casi.ntrs.nasa.gov/19790012790_1979012790.pdf
The above 373 Page report, available for download is called:
Sun, Weather and Climate.
It is full of solar wind/cosmic ray info and weather correlations from 1978.
Hope you can use it for some fuel for some future work.
Max

pat
December 23, 2010 9:07 pm

Why is the ‘energy’ missing? Because these people are dolts. Much like those unlikely fools that argue the missing ‘energy’ is absent from the first 2,000 feet of surface ocean but is stored in the next 13,000 feet.
It is as clear as day that during the last warming period that life forms sucked up immense amounts of CO2 and used many other biologic resources for propagation and growth. It is equally clear that the insulating effect of CO2 is hugely over-exaggerated. I would suggest that Gaea, if such a being had reason, would thank humans for releasing CO2 just at the time when it was most useful. ie, a warm period.

Darren Parker
December 23, 2010 9:08 pm

I’m amazaed that how much common sense , stuff that we all take for granted if we just went outside an observed weather, is finally being admitted to in the modelling. It’s obvious also that the Northern and Southern Hemispheres hold and release energy at different rates due to the land/water ratios.

savethesharks
December 23, 2010 9:22 pm

Excellent post Willis. “Negative climate sensitivity”. Now there’s an incendiary term.
Chris
Norfolk, VA, USA

Charles Higley
December 23, 2010 9:33 pm

Even if the GHGs did cause some warming of the lower troposphere, it would only serve to
(1) increase evaporation, as described above, and
(2) make the moist warm air more ready to ascend causing convection.
The result would be to rev up this heat engine and carry energy to altitude sooner and more rapidly.
It’s a win-win for a negative feedback.

December 23, 2010 9:34 pm

Your conclusion seems to dovetail with the one that Ferenc Miskolczi made a few years back — i.e., that the greenhouse effect has a ceiling, and that limit must have been reached already — naturally.

Charles Higley
December 23, 2010 9:36 pm

It is all of the common sense stuff, such as the basic water cycle Mr. (Dr.?) Eschenbach described above that the “climate scientists” want us to ignore (as they do). Instead, they want us to think that climate science is much too complicated for us (other scientists and ignorants) to understand.

SidViscous
December 23, 2010 10:06 pm

vis-a-vis the missing energy.
Has anyone looked down the back of the couch?

Keith Minto
December 23, 2010 10:27 pm

Good post, as usual Willis.
I am keeping a low profile from the pre-Christmas noise but there is a problem that has bothered me for a while. Looking at item 2. of figure 1. Note that all of the energy goes into evaporating the molecule of water……….
The transport of the heat to the upper troposphere takes place in the thunderstorm.
Some of the heat, but all of the heat?
The assumption is that the water vapour molecule, being less dense than the surrounding atmosphere (density ratio water vapour/air @ STP kg/m^3, 0.595/1.275) will rise, solo, and join with other consenting H2O molecules when the correct STP is reached.
But what if the water molecule released at the sea surface combines lower down in the troposphere and releases its latent heat there ? I do not mean droplet formation as in a cloud but unseen micro droplets, releasing latent heat lower down, condensing on dust particles, salt particles and dimethyl sulphide.
These nuclei are there, what is to stop this energy release lower down from the cloud base? it is hardly a pure, pristine environment from sea surface to cloud base.
Is this reasonable? It may be a source of lower atmosphere heat.

anna v
December 23, 2010 10:47 pm

Keith Minto says:
December 23, 2010 at 10:27 pm
These nuclei are there, what is to stop this energy release lower down from the cloud base? it is hardly a pure, pristine environment from sea surface to cloud base.
Is this reasonable? It may be a source of lower atmosphere heat.

Hot air rises so the energy will go up following the storm upward motion in any case.
There was a long thread in Lucia’s and The Air Vent where a new model of wind creation is proposed by Anastassia Makarieva et al who say that condensation in addition to releasing energy creates a vacuum and winds come in to fill it up and go up, creating the large winds seen in cyclones. Not many people were convinced but in either model hot air ends up high releasing energy.

Rod Gill
December 23, 2010 10:57 pm

But what if the water molecule released at the sea surface combines lower down in the troposphere and releases its latent heat there ?
My reading of this is that here is just one example of where the equations do not work at all, because climate sensitivty in this case is zero. So what about the average sensitivity? Who cares?! Clearly sensitivity does change with temperature and circumstances. If models can’t even handle what happens at night and changing cloud levels an types, the model in no way represents even an “average” scenario.
Since thunderstorms are a significant pump of heat from sea surface to space in the tropics, they need to be modelled correctly. And if some of the time the sensitivity is negative? Well as Willis says, that’s another story!

Ashby
December 23, 2010 11:01 pm

If your thought experiment is correct, wouldn’t the effect of increased co2 be increased water transport rather than a hotter globe e.g. more rain and snow?
That would seem to be a net positive. At least from LA and other semi arid environs.

Gino
December 23, 2010 11:32 pm

So is “climate sensitivity” just a way to express specific heat for the atmosphere?

AlanG
December 23, 2010 11:45 pm

I feel strongly you’re on the right track, Willis. I hope you develop your ideas fully. I think you’re agreeing with Spencer and Lindzen. Many aspects of the climate are highly non-linear. Feedback from water vapor starts out as strongly positive and ends up as strongly negative when the thunderstorms get going. But there is something else you should consider – MASS. Rising (and descending) air masses have momentum (mv) and kinetic energy (1/2mv^2). The momentum ensures that thunderstorms take time to form and run on longer than they would if air had no mass. Also, the air rises rapidly in thunderclouds and descends slowly outside of them so the kinetic energy of the air is a net carrier of energy aloft because KE is proportional to the square of the velocity. That will account for some more of your missing energy.
Non- linearity also appears in temperature because of the Stefan–Boltzmann law (sigma * T^4). Take two conditions where the average temperature is 15C (288K):
1) The temperature spends half the time at 278K and half at 298K (+/-10K).
2) The temperature spends half the time at 283K and half at 293K (+/- 5K).
The average is 288K in both cases but the LR energy emitted is different:
1) 298^4 – 278^4 = sigma * 1,913,333,760 = 107.2232239 J/s/m2
2) 293^4 – 283^4 = sigma * 955,802,880 = 53.5631934 J/s/m2
That’s a big difference. This is why all models and formulae based upon static averages are useless.

Paul Loock
December 23, 2010 11:53 pm

Brian H :
“heat pipe” is the best short description of what happens on a water planet when one end of the system (the surface) is heated by radiation.

Keith Minto
December 23, 2010 11:55 pm

anna v says:
December 23, 2010 at 10:47 pm
True, but radiative heat loss to space may be reduced if condensation in part occurs lower down, changing the equations. The condensing radiative air is caught between the sea surface and the cloud base.
I did not see the Lucia’s thread but I will check, thanks,(9mth old on my knee, thrashing!!!!)
NS 27 Nov has an article Cloud Power (p40) that has a model proposed by Quinn Brewster that suggests vapour/water phase change is radiative at least in part. (I thought it was?)

AlanG
December 24, 2010 12:10 am

Oops. The LR energy emmited in the 2 examples are:
1) average of S *298^4 and S * 278^4 = 388.3282574 W/m2
2) average of S * 293^4 and S * 283^4 = 386.2360502 W/m2
That’s a difference of 2.09 watts

michel
December 24, 2010 12:10 am

“Climate sensitivity is not a constant, it is a function of temperature”
Yes, this is probably the fundamental factor. We don’t yet seem to understand feedbacks, but this seems very plausible.

MikeA
December 24, 2010 12:12 am

Willis, my own speculation had been that the ocean had been absorbing the heat via the thermohaline currents, however your model is provable by observation. Can you suggest an observation to prove your model?

NovaReason
December 24, 2010 12:34 am

Keith Minto says:
December 23, 2010 at 11:55 pm
What I get from your post is that you’re suggesting a non-negligible heat loss caused by condensation of water vapor in the lower troposphere, but as anna v attempted to explain, the release of energy due to condensation would warm the surrounding air, which would then continue to rise (as hot air is wont to do) following the air currents of the uncondensed water vapor.
However, some heat *is* trapped in the lower altitudes (remember, GHGs are real, water vapor *is* one, and carries most of the heat in our atmosphere, much more than the paltry amount absorbed and held by CO2), and this could be a mechanism behind it.
My question to you then, sir, is as follows. “What kind of effect would that have on the climate of Earth?” Are you suggesting some bold new form of GHG trapping? This seems actually rather inconsequential… we know that thunderstorms form, and we know they form by the travel upwards of water vapor, which then condenses and allows for heat escape to space… some of them not making it wouldn’t invalidate Willis’ argument… if you’re suggesting that’s where the “missing energy” went, then you need to brush up on the laws of thermodynamics again, because for any significant amount of energy to be lost to this mechanism would requires substantial, noticeable temperature increase in the lower troposphere (read as, Global Warming gone nuts). Furthermore, it would increase global warming not based on CO2 levels, but levels of dust and particulates in the air, things not TYPICALLY associated directly with global warming. Increased dust levels leading to higher tropical temperatures… haven’t seen the study yet.

December 24, 2010 12:39 am

Willis,
I think this is correct “At the top of the troposphere, where it leaves the thunderstorm, there is little atmosphere of any kind. From there it is free to radiate to space with little interference.” Not much water vapor in the stratosphere and the pressure is low so CO2 absorption lines don’t have much pressure broadening. Lacis reckons that why Mars doesn’t have strong “greenhouse” effect even though there’s lots and lots and LOTS of CO2.
anna v, I reckon the onset of convection and inrushing air at the surface gaining rotation and causing tropical cyclones is indisputable. Rising air causes low pressure at the surface just as subsiding air causes high pressure at the surface. What the heck do these people think causes dust devils, tornados and cyclones?????
Some of these people need to get out more and start flying around in the atmosphere for a few years.

steveta_uk
December 24, 2010 1:09 am

Perhaps I’m being thick, but I don’t think equations 1 and 2 contradict.
What you seem to have is a very simplified model where dQ in = dQ out, with dT = 0.
So dO = dT / S = 0/S = 0.
For no energy lost, eq 2 becomes 0 = 0 / S
Which by my reading means sensitivity is irrelevant in this simple model, not that it is 0.

Caleb
December 24, 2010 2:00 am

RE: MikeA says:
December 24, 2010 at 12:12 am
The best way to avoid all the beautiful complexity of the earth’s systems of energy-balancing is to stand outside the system, and to devise some method of pointing a satellite at the sun to measure the incoming energy, and pointing a second satellite at the earth to measure the outgoing energy. That way you don’t have to enter the engrossing and fascinating subject of what the energy does while it is here.
I think thermohaline circulation is one of the most under-funded and under-studied elements of the climate system. When you consider the “input” to the thermohaline currents undergo huge fluctuations, due to the difference between the seasons when ice is forming and the seasons when ice is melting, it makes sense that the current itself might reflect those fluctuations in some manner. Perhaps it throbs like a slow heart beat, with waves traveling along the thermocline. Then, perhaps, these fluctuations reappear at the surface at points where upwelling acts as the “output” of the thermohaline circulation. (Most focus is upon winds, as a cause of upwelling, and very little attention is focused on rate-of-flow.)
Perhaps, perhaps, perhaps. The fact remains, there is still much we don’t know.
What irks me about the likes of Hansen is their know-it-all attitude. Their brains are stuck in some sort of preconceived notion, and seem to atrophy rather than to seek further. Worse, they actively resist research dollars going to study things such as thermohaline circulation, claiming their computer models deserve it all.
The idea that thunderstorms serve as a safety-valve, releasing extra heat from the energy system, has always seemed a very obvious solution to me. Therefore Hansen must have a stock answer, to deflect research dollars back to his computer models, when fellows like Willis Eschenbach ask the blatantly obvious questions.
However all these fascinating areas of research are based on following the energy about once it arrives here on earth, and shifts from state to state, both via changes in latent energy and also via changing from the energy of a sunbeam to the energy involved in transporting massive amounts of water about. This complexity can be avoided by stepping outside the system, and measuring energy coming in and energy going out, via satellite.

Stephen Wilde
December 24, 2010 2:15 am

Willis said:
“My point is that there is an energy path that completely dodges the surface. It comes in and just evaporates water. It does not raise the surface temperature at all. This gives a climate sensitivity of zero.”
This is a point that I have been pushing for some time in connection with the so called back radiation (or extra downward IR) from more CO2 in the air.
I’ve started from the position that it is for AGW proponents to demonstrate that there is any surplus energy left over from the extra IR once the enhanced evaporation has occurred given that evaporation is a net cooling process.
After loads of bluster and disbelief I have never seen it demonstrated, merely assumed.
So on that basis more CO2 cannot affect ocean temperatures yet it is ocean temperatures that control air temperatures. I see that as quite an obstacle for AGW to overcome.
So what does happen to the extra energy once converted to water vapour via enhanced evaporation ?
Just a miniscule unmeasurable and inconsequential increase in the speed and/or intensity of the water cycle.
Given that solar and oceanic effects on the speed of the water cycle seem to have the ability to shift the jet streams many hundreds of miles latitudinally and cause them to wave about meridionally I suggest that human CO2 might just shift the jets a few yards or so.
Not what I would call catastrophic.

December 24, 2010 3:23 am

The energy is missing, because it was not trapped by “enhanced greenhouse effect”. Lindzen&Choi and Spencer&Braswell presented by real life satellite data, that OLR is not being reduced as the models predict, basically because the alleged increase in tropospheric water vapor as secondary effect is missing, so total IR optical thickness did not change much.

Stephen Wilde
December 24, 2010 3:26 am

Caleb said:
“I think thermohaline circulation is one of the most under-funded and under-studied elements of the climate system. When you consider the “input” to the thermohaline currents undergo huge fluctuations, due to the difference between the seasons when ice is forming and the seasons when ice is melting, it makes sense that the current itself might reflect those fluctuations in some manner. Perhaps it throbs like a slow heart beat, with waves traveling along the thermocline. Then, perhaps, these fluctuations reappear at the surface at points where upwelling acts as the “output” of the thermohaline circulation.”
I agree and have proposed just such a scenario to account for part of the 1000 year climate cycling we see from Roman Warm Period (and earlier) to date given that the thermohaline coirculation takes around 1000 years. The other part being solar induced.
The solar cycling is similar at present and approximately in phase with the THC cycling but there is no reason why they should always be in phase as I have discussed elsewhere.
Proposing that underlying THC cycle also has the potential to deal nicely with CO2 observations on the basis that upwelling from warmer THC waters created during the MWP would now be reducing the CO2 absorption capabilities of the oceans so as to at least partly produce the current steady CO2 rise which seems to be unrelated to anything else going on in the climate at present.
So, a lot to play for with regard to the THC, as you say.

Stephen Wilde
December 24, 2010 3:34 am

“The idea that thunderstorms serve as a safety-valve, releasing extra heat from the energy system, has always seemed a very obvious solution to me.”
Take one more step and propose that the entire water cycle globally serves as a safety valve. The phase changes of water being the primary energy shifting mechanism.
Or rather some sort of modulator which always reacts in a negative fashion to any forcing either internal or external to the system in order to retain or regain temperature equivalence between sea surface and surface air temperatures.
That basic temperature being dictated by density and pressure differentials at the point of contact between sea and atmosphere plus solar energy input to the system at any given moment.
CO2 could then only have an effect if it significantly altered density and pressure differentials but it does not.

December 24, 2010 3:43 am

Another great post, Willis, and thanks. Science in high school was never this interesting.

Joe Lalonde
December 24, 2010 4:10 am

Willis,
I enjoy looking at the research and analysis you do. They are very good.
But the complexity of energy if far more complex then simple equations that you illustrate. CO2 does have some effect of blocking what would normally have been energy hitting the oceans to be absorbed/deflected. Add to this the surface salinity changes and the normal balance is totally changed into the cooling effect of oceans when also adding in cloudcover.
H2O is compressed gases that are forgotten as we are used to water being it’s own element.
Rotational energy is the planet pulling the atmosphere generating the pressurized environment and energy of up to 1669.8km/hr at the equator.
Rotational energy is already infused into the atmosphere and is the physical energy available for wind.
In current science, our planet needs not to rotate to produce the current theories.

Joe Lalonde
December 24, 2010 4:19 am

Willis,
Penetration of solar radiation in the oceans is not very far into the oceans due to the planetary movement of rotation bending the energy in a semi-solid state.
On shore lines, the penetration is absorbed by the material under the shallow water to retain partial heat. Again it is only so far before the peneration is reflected back.
Ocean density becomes a factor as the deeper you go.

Lars P
December 24, 2010 4:22 am

Thanks for the excellent post Willis. I think this clarifies part of the energy path and explains why models ignoring evaporation are flawed.
Not being an expert I prefer to keep a low profile and follow-up closely the AGW discussion as it has too big implications to ignore. Sadly it is rather difficult to find unbiased sources of information, this is why I want to thank all contributors for the effort and the high scientific debate one can find here.
Merry Christmas!

Schrodinger's Cat
December 24, 2010 5:05 am

IR radiation from the GHG will be absorbed by the water molecules at the surface of the sea and the energy will cause the excitation of these molecules. They will vibrate (stretch, bend, etc) more vigorously, thus converting IR energy to kinetic energy. The more energetic of these molecules will escape into the atmosphere as evaporated water. The IR cannot penetrate the sea to any depth because water is effectively opaque to IR. In my view the greenhouse effect increases evaporation from the sea surface rather than raise its temperature.

Barry Day
December 24, 2010 5:12 am

What about the “extra energy” from below the Ocean surface
http://au.myspace.com/my/photos/album/965488/custom/list?u=1
Troubling increase in global volcanic activity,REMBERING ONE LAND VOLC’S = 85 SUBMARINE VOLC’S X 20 % ? INCREASE IN VOLCANO’S SINCE 1970.
Underwater volcanoes
Therefore, underwater volcanism should also be the greatest in 500 years. … In the U.S., precipitation has increased 20 percent just since 1970. …
http://www.iceagenow.com/ocean_warming.htm
http://australianconservative.com/2010/11/troubling-increase-in-global-volcanic-activity/
In his book, “Not by Fire but by Ice” the foremost authority on ice ages and magnetic reversals, Robert W. Felix, quoted Peter Vogt of the U.S. Naval Oceanographic Office who warned that,
XXXXXXXXXXXXXXXXXXXXXXXXXXXXXXXXXXXXXXXXXXXXXXXXXXXXXX
“Almost all tectonic movement can be linked to magnetic reversals. Seafloor spreading, sea level changes, mountain growth, earthquakes, and volcanism all seem to speed up whenever the frequency of reversals speeds up.”
XXXXXXXXXXXXXXXXXXXXXXXXXXXXXXXXXXXXXXXXXXXXXXXXXXXXXX
Magnetic reversals are part of the cycles scientists have determined existed over the 4.5 billion years of the Earth’s existence. They range from the most ancient, the Devonian, to the Holocene, from 10,000 years ago to the present. “At least twelve (magnetic) reversals can be linked to extinctions and climatic deterioration during the last three million years alone,” says Felix.

DocMartyn
December 24, 2010 5:17 am

people often say that people today do not know what real work is. Some 990 millimetres (39 in) of fresh water falls on each unit area of the Earth each year; some 70% of it having been distilled from salt water. This water was carried against a gravity well as high as 30,000 feet. When it made the gas/solid transition it dumped a lot of IR radiation at high altitude and caused a huge pressure/temperature drop.
Do you have any idea the amount of work is involved in the water cycle?

John Day
December 24, 2010 5:36 am

@Willis Eschenbach
> PS – Please don’t say “but you left out the greenhouse gases”.
But you didn’t leave out the most important gas (aka ‘control knob’) => water vapor.
CO2, by itself, doesn’t have much effect, as we can see on Mars (where it is almost 30 times more abundant per unit of surface area than on Earth): the mean surface temperature and black body temperature are about the same, ~210K.
CO2 does have voracious appetite for IR radiation in the main terrestrial radiation band around the 15 micron wavelength. So, on Earth, surface heat generated by insolation is totally absorbed in that main band within the first few hundred meters of the troposphere.
But this extra heat energy doesn’t just sit there, it tends to create rising convection currents which just makes your heat engine work faster, making the climate sensitivity smaller than than it would in a static environment.

December 24, 2010 6:08 am

You are on the right track. The next step is to show how energy flows from the tropic to the poles in the form of water vapor and clouds and how ice at the poles insulates and causes the earth to act as a capacitor saving energy. At the North pole, the annual freeze/thaw cycle serves as a pump stroke keeping the deep ocean currents going. That’s a lot of kinetic energy. As an aside, CO2 goes along for the ride and has no measurable effect on this constantly changing energy exchange. http://www.kidswincom.net/climate.pdf and http://www.kidswincom.net/CO2OLR.pdf.

David
December 24, 2010 6:19 am

Willis
Willis, is it not possible that some systems receive energy quickly, but manifest it slowly? While riding a bicycle up a steep hill I can quickly increase the energy in my legs, which, if I am in a low gear will efficiently manifest as a change in speed, but if I am in a high gear this speed change will manifest far slower. Do we really have the understanding and sensitivity in all of our measuring to capture the energy budget as it changes form, phase, and location, or are there possibly slow changes in thermocline depths, hydrologic cycle speeds, atmospheric elevations, large ocean currents etc, that can receive energy quickly but manifest it as temperature slowly or even imperceptibly in regard to our ability to capture these changes?
Also have you looked at the annual cycles in all that you are measuring to see if the earth’s seasonal energy pulse can reveal some of this mystery? Sunlight, falling on the Earth when it’s about 3,000,000 miles closer to the sun in January, is about 7% more intense than in July. Because the Northern Hemisphere has more land which heats easier then water most people state that the Earth’s average temperature is about 4 degrees F higher in July than January, when in fact they should be stating that the ATMOSPHERE is 4 degrees higher in July. In January this extra SW energy is being pumped into the oceans where the “residence time” within the Earth’s ocean land and atmosphere is the longest
As these immense changes in SWR TSI happen annually, then how much and how rapidly changes in those things you measured in Figure 1 and figure 2 match these annual changes, as well as changes in albedo and cloud cover should give deeper insight relative to heat and energy flux within our earth system.
Sunlight, falling on the Earth when it’s about 3,000,000 miles closer to the sun in January, is about 7% more intense than in July. Because the Northern Hemisphere has more land which heats easier then water most people state that the Earth’s average temperature is about 4 degrees F higher in July than January, when in fact they should be stating that the ATMOSPHERE is 4 degrees higher in July. In January this extra SW energy is being pumped into the oceans where the “residence time” within the Earth’s ocean land and atmosphere is the longest. Some of the energy increases the mean evaporation rate of the earth and is transmitted to latent heat before being released, but most of the extra energy is lost to the atmosphere for a time as it is contained within the ocean, but eventually it is radiated from the ocean as LWR and increased latent heat in water vapor.
In actuality, due to the more intense southern sun’s SW radiation being trapped in the oceans, the earth is gaining energy in the southern hemisphere summers, and losing energy in the northern hemisphere summer. The extra energy is just hidden in the oceans for a time while the northern hemisphere summer reacts on a more sensitive atmosphere, creating a rise in atmospheric heat, but a net loss to the planet relative to the southern hemisphere summer.

Dave Springer
December 24, 2010 6:21 am

AJStrata says:
December 23, 2010 at 8:41 pm
“There are two very large and very dynamic energy sources (external energy from the Sun and internal energy from our molten core).”
Energy from the core neglible. It amounts to a few milliwatts per square meter. Compare that to energy from the sun which is a few hundred watts per square meter. The energy coming in from the sun is 100,000 times greater than the energy upwelling from the core of the earth. The reason is simple enough – rocks are excellent insulators.

David
December 24, 2010 6:21 am

Excuse the double paste in the last paragraph please.

1DandyTroll
December 24, 2010 6:32 am

I might have the answer. After reading the above post, I inclined to think all that missing energy is pent up in all the climate hippies and that’s why they’re so up tight and so tense they’re struggling not to go ballistic.
Remember that smoke trail over California a while back, even though no rocket fuel what so ever where ever found to have been gone missing, or miss-appropriated, and everything sold was accounted for? Aaah-Ha!

Dave in Delaware
December 24, 2010 6:37 am

* The ‘tiny bit of energy’ in steps 1 and 2 of your figure fits with long wave IR ( back radiation), which would be absorbed immediately at the surface. (Inbound shorter wave radiant energy (from the sun) is more likely to penetrate deeper below the surface.)
* If we take a step earlier in the process (before your Step 1), the initial LW IR sent from the warm surface toward the atmosphere is absorbed by H2O and CO2, and most of that is converted to kinetic energy (temperature) through collisions with air molecules.
So …. the initial IR transmitted up from the liquid surface is absorbed and warms the air above the surface, kick starting the upward convection as described by anna v and others. And any long wave IR back radiation is absorbed at the surface, vaporizes a bit of water which caries that energy as latent heat up with the already rising warm air as described by Willis. The result, as described by Stephen Wilde is –
“Just a miniscule unmeasurable and inconsequential increase in the speed and/or intensity of the water cycle.”

December 24, 2010 6:38 am

BTW,
I should have added that the above model is one small system the systems of ‘captured’ of energy I was attempting to address. It is accurate and does show how energy is consumed without effecting temperature or climate. There are systems within even this one. The energy to evaporate a water molecule (or any of elements and compounds in the oceans) is simply the start of the energy consumption. Then there is the build up of potential energy as wind and thermal energy raise that molecule up into the upper atmosphere. When the molecule condenses it loses some of its energy, but the wind can keep it aloft until sufficient mass develops to create a rain drop. In between the water vapor can freeze, liquefy or return to gas – sometimes many times over.
When we have high humidity we have lots of the energy stored in the atmosphere. But there is still lots of water stored in the oceans and in the land. High humidity means a different climate response for that region of the globe.
And the journey of a water molecule is just one mechanism. What about the wave actions and the pounding of shores – which consume energy. The complexities of volcanic eruptions and dust particles which change the absorptive nature of the atmosphere.
The number and intensity of all these individual physical processes divert and consume and release energy into the overall energy balance I mentioned at the top of this post. And our confidence (error bars) on these processes means we do not have the knowledge and experience to know the world temperature today within a degree – let alone 50-100 years ago. And forget 100-1000 years in the past or future.
Flip a coin, it would be just as accurate.

E.M.Smith
Editor
December 24, 2010 7:02 am

Willis, while I find these posts well written and interesting, I also feel I ought to point out that we’re sitting on top of a 6 × 10^21 ton mass of very “hot stuff” that is a large nuclear reactor filled with molten iron.
Measuring the temperature flitting back and forth in the wisp of air clinging to the surface seems to be ignoring a very large elephant mass in the center of the “room”…
While temperatures in the soil tend to stabilize several feet down, the top layer is not a constant temperature. Take a spherical shell the size of the earth and about a meter thick. What’s the heat content of a 5 C temperature change?… All that rain water does not just sit on the surface. A lot of it enters the soil and takes it’s heat with it…

Pamela Gray
December 24, 2010 7:10 am

I prefer to come at it from the perspective of anthropogenic CO2 increased LW radiation coming back to heat us (IE water, air, land). This extra heat can be calculated. However, since scientists can’t find it here on Earth, how is it being released back to space? First off, the extra heat from anthropogenic CO2 is quite small compared to what naturally occurs with greenhouse gases teleconnecting with natural oceanic and atmospheric conditions. The only thing we need to model then is how the various systems are handling (and apparently getting rid of) this tiny bit of extra heat, not how it handles any and all heat.

Dave Springer
December 24, 2010 7:11 am

Stephen Wilde says:
December 24, 2010 at 2:15 am
“This is a point that I have been pushing for some time in connection with the so called back radiation (or extra downward IR) from more CO2 in the air.”
Yeah, that’s a sticking point for me too. IR is absorbed by water in a vanishingly thin surface layer which results in increased evaporation but no heating of the water. Visible light on the other hand penetrates hundreds of meters and does in fact heat the water. So called back radiation then cannot heat the water but in fact heats the troposphere at the cloud layer. The AGW boffins know this. It’s why their model of non-condensing GHG heating predicted a signature effect of more heating higher up in the troposphere than at the surface – the infamous tropospheric hot spot.
That’s all well and good and it does work that way but that way results in rather little surface heating. Therefore the boffins introduced a positive feedback effect from water vapor. They reason the increased evaporation rate raises the relative humidity of the air and it’s well known that humid air keeps things warmer than dry air. Thus the small surface temperature rise from more NCGHGs (CO2, methane, et al) fosters a rise in the condensing GHG (water vapor). If things actually worked that way we’d have a runaway greenhouse. But here’s the kicker. One of the more well established paleo-atmospheric facts that must be considered is that for most of the time in the past 500 million years the atmosphere held 10 to 20 times more CO2 than the present atmosphere and there was no runaway greenhouse. Global average temperature was at most were 8 degrees C warmer than today and moreover the higher temperatures were not evenly distributed but rather concentrated in higher latitudes with the effect of turning the earth green from pole to pole. That is the “normal” state of the earth’s climate. What we have today is a temporary, still rather cold, and long in tooth intermission in the full blown ice age that began about 3 million years ago.
Where I believe the AGW boffins go wrong is that there is no increase in relative humidity caused by increasing GHGs but rather an increase in the speed of the water cycle. RH remains constant there’s just more and faster transport of water vapor from surface to cloud layer and more rain. The carbon cycle speeds up too as the higher CO2 level and milder temperatures in the wintry north accelerates and expands plant growth which means more animals get fed and when they die more bacteria decompose them releasing their stored carbon back into the air which is then food for plants and the cycle continues.
Increasing CO2 is a good thing. Increasing warmth is a good thing. The only downside is rising sea level as it pretty much goes without question that as the wintry north gets warmer glaciers will melt but that process takes thousands of years. Wild terrestiral plants and animals have plenty of time to migrate away from the encroaching sea and (like it or not) humans will have to do the same. The net benefit to the biosphere as a whole though is tremendous as the whole world turns a lush verdant green.

Pamela Gray
December 24, 2010 7:16 am

What I mean to say is that we don’t have extra energy entering the system. CO2 does not cause extra energy to enter the system, unless we are saying that anthropogenic CO2 is revving up the Sun. AGW says that we have some of the constant incoming heat energy staying around instead of cycling through a leaky system. But since we can’t find it here, it must be leaving in a way that AGW has not been able to model.

Steve Keohane
December 24, 2010 7:20 am

Willis, the comment I made in your last thread wrt missing energy seems more apropos after this piece. That was that the atmospheric height has decreased. It was published in the AGU, http://www.agu.org/news/press/pr_archives/2010/2010-28.shtml . Allegedly, the thermosphere, between 55 & 300 miles, had lost 30% of its density by 2009 due to reduction in the solar wind. I came across a reference I can no longer find this year that put the contraction at 125 miles. To me, this would allow for a more rapid transfer of energy to space during times of low solar activity, slightly less energy into the system, but more going out exacerbating the minimal change in TSI.

Joel Shore
December 24, 2010 7:32 am

Willis,
Follow your reasoning to its logical conclusion: In order to send more heat back into space, the upper troposphere must warm, since the only way it exchanges heat with space is via radiation and radiation depends on the temperature. So, what you are suggesting is that the upper troposphere in the tropics warms more than the surface.
However, to the extent this is true, it is already included in the models as a negative feedback called the “lapse rate feedback”. Now, I suppose you could argue that they are underestimating the lapse rate feedback…but then that would mean that the tropical tropospheric amplification should be larger than the models predict. And yet, most empirical data currently suggest that it is the same or smaller than the models predict. The data suggesting smaller is pounced upon by skeptics as evidence that the models are wrong (and sometimes wrongly claimed to show that the warming that is occurring can’t be due to the mechanism of greenhouse gases). While there is reason to believe that the long-term trends in the upper tropospheric balloon and satellite data has some artifacts, you now seem to want to claim that the actual temperature trend in the upper troposphere would be larger than the models predict.

Steve Keohane
December 24, 2010 7:35 am

I meant to add this from the AGU abstract, “The results showed the thermosphere cooling in 2008 by 41 kelvins (about 74 degrees Fahrenheit) compared to 1996, with just 2 K attributable to the carbon dioxide increase.”
CO2 causes cooling?! WUWT?

Pamela Gray
December 24, 2010 7:36 am

I can’t see how this tiny bit more heat (anthropogenic) at the surface will have much of an affect on land. It won’t cause much of a difference in soil evaporation. It could be absorbed in the greening of the planet. Plus pressure differences will sweep some of it away to other locations. If it landed over areas experiencing strong radiative cooling, up it goes. Some of it would be taken out over the oceans. This would cause a bit more evaporation at the water’s surface but the tiny extra amount of evaporation would likely just be added to the already in motion hydrological cycle. It may just come down to the jets taking care of this tiny amount of heat, calculated by the AGW models to stay here, by moving it around to places that already have atmospheric escape hatches doing their thing. And since adding more CO2 has diminishing affects, I think the Earth’s system are dealing with it just fine. The reason why we can’t find it is because it isn’t here in amounts outside the standard error.

harrywr2
December 24, 2010 7:39 am

Caleb says:
December 24, 2010 at 2:00 am
“The best way to avoid all the beautiful complexity of the earth’s systems of energy-balancing is to stand outside the system, and to devise some method of pointing a satellite at the sun to measure the incoming energy, and pointing a second satellite at the earth to measure the outgoing energy. ”
We have that and we still end up with Trenberth’s ‘missing heat’. In the catastrophic scenario we have 1/2 W/m2 hiding somewhere building up over decades that will suddenly stop hiding and kill us all, similar to the Loch Ness monster.
Without heat hiding somewhere in the system the observed surface temperature increase over the last 100 years is pretty close to what the physics says. I.E. A doubling of CO2 will cause a 1.2 degree C rise in global temperature all other things remaining equal, of which .8 degrees C is already ‘water under the bridge’ so to speak.
Of course the missing 1/2 w/m2 out of 1366 could be measurement error in the satellites that measure incoming/outgoing radiation or the argo buoy’s that measure ocean temps or the satellites that measure sea level.

Pamela Gray
December 24, 2010 7:52 am

Dave, I am thinking the only change I would make to your statement (which is on the whole entirely well thought out) is that our land masses have changed positions, re-directing currents and their teleconnected atmospheric systems in such a way as to freeze up the poles (of particular note the AO and AAO atmospheric systems, along with the Antarctic circumpolar current). I don’t think CO2 increases would green up the poles unless land location changes redirected oceanic currents.

Pamela Gray
December 24, 2010 7:53 am

By the way Willis, I love, love, love this thread. Thanks for starting it.

RC Saumarez
December 24, 2010 7:58 am

The climate is an open thermodynamic system as it has energy flowing through it. The missing heat cannot be considered without considering the energy required to keep the climate in its negentropic (i.e.: non-equilibrium) state. In my view any analysis of heat fluxes through the climate that ingores entropy must be wrong.

E.M.Smith
Editor
December 24, 2010 7:59 am

Oh, and “permafrost” tells you how much ground freezes / thaws each year.
Small map here:
http://nsidc.org/fgdc/maps/ipa_browse.html
Data about it here:
http://nsidc.org/data/ggd318.html
The main map is here with full size readable legend.:
http://nsidc.org/data/docs/fgdc/ggd318_map_circumarctic/index.html
and in the legend it says percent 10 – 20 meters down that’s frozen. So that says up to 20 m of depth CAN freeze / thaw each year. That’s going to hold a lot of heat…
Oh, and all the places NEAR the permafrost that does not show on the map but gets snow each winter will also freeze “to some depth” that varies with the place.
So there is a very large chunk of heat flux into and out of the earth itself each freeze / thaw cycle. I’d not ignore the frozen parts of the world and the dirty parts of the world…

David
December 24, 2010 8:03 am

harrywr2 says:
Regarding December 24, 2010 at 7:39 am
Caleb says:
December 24, 2010 at 2:00 am

Without heat hiding somewhere in the system the observed surface temperature increase over the last 100 years is pretty close to what the physics says. I.E. A doubling of CO2 will cause a 1.2 degree C rise in global temperature all other things remaining equal, of which .8 degrees C is already ‘water under the bridge’ so to speak.”
The vast majority of that rise in CO2 is only for the last 30 years, or 1/2 of a 60 year cycle. The 1.2 C rise only reflects in those thirty years, which is matched by previous natural, (non CO@ induced) rises. Give us another 10 to twenty years and we shall see.

Pamela Gray
December 24, 2010 8:05 am

harrywr2, I believe Trenberth’s conclusion that AGW heat is hiding somewhere is partially based on a very short span of data actually measuring incoming and outgoing radiation added to a model. We do not have a 100 year’s worth, and it really should be 120 year’s worth, of this actual satellite data. Why do we need one longer than what we have? Because some of our natural oscillating cycles that have measurable affects on incoming and outgoing radiation have cycles of 60 years or more. Double that and you get a 120 year minimum to see just one or two cycles. Not exactly a sampling rate I would want to hang my hat on.

Pamela Gray
December 24, 2010 8:11 am

E.M. Smith, this past summer NE Oregon demonstrated consistently cooler soil temperatures at worm depth. I know. I dug into that soil looking for fishing worms. Worst year I can remember for gathering worms and coldest soil I have touched in my 54 year memory. Usually, after a rain, huge nightcrawlers could be picked up by the handfulls. I did not find a single nightcrawler on the road this last summer. Veggie gardens around here also suffered due to this rather cool soil. It is my opinion that the Earth underneath didn’t cause this, it was the depth of the winter freeze combined with very cool summer temps and cold cold rain whut dun it.

James Macdonald
December 24, 2010 8:12 am

Willis—Your type of thinking and research is what meteorologists, like myself, love.
We observe what you postulate.

E.M.Smith
Editor
December 24, 2010 8:28 am

Dave Springer says: The net benefit to the biosphere as a whole though is tremendous as the whole world turns a lush verdant green.
Well, in an extreme case, but using the IPCC predictions projections it’s not very much greener….
http://chiefio.wordpress.com/2010/12/12/what-me-worry/
has the Koppen-Geiger maps of the world now and as the IPCC “reimagines” it.
You can see differences… if you squint enough and get really really close to the screen 😉
Frankly, I don’t see a darned thing to worry about even IF AGW is happening. IFF we had AGW, it certainly isn’t Catastrophic AGW. So CAGW is just incredibly bogus, even if AGW were real, and AGW looks to be an overblown fantasy.
I really do recommend a look at those graphs. Biggest changes I could spot were more hospitable crop land in southern Canada and Russia, a bit nicer Ukraine and Belarus, and more of the Pacific Northwest gets a bit of the Mediterranian Climate that folks all over the world seem to love.
It looks like Chile might get a bit nicer, too. And Southern Alaska warms up a great deal to being almost as warm as Fargo North Dakota 😉
(For folks not familiar with it, Fargo is often the coldest darned part of the “lower 48” in any given winter and has become a bit of a metaphor for “frozen bippy”…) “Bippy” being a ’70s metaphor for “sit upon”… “sit upon” being a euphamism for… but I digress 😉

Pamela Gray
December 24, 2010 8:39 am

I’ll bet my Meacham, Oregon pass against your Fargo.

Pamela Gray
December 24, 2010 8:42 am

Here are some of Oregon’s Arctic Queens
http://www.oregonphotos.com/Meacham.html

Joel Shore
December 24, 2010 8:43 am

Steven Keohane says:

I meant to add this from the AGU abstract, “The results showed the thermosphere cooling in 2008 by 41 kelvins (about 74 degrees Fahrenheit) compared to 1996, with just 2 K attributable to the carbon dioxide increase.”
CO2 causes cooling?! WUWT?

Yes…The increase in greenhouse gases is expected to cause cooling of the upper parts of the atmosphere (the stratosphere and apparently the mesosphere and thermosphere too). The different temperature change structure expected in the vertical from greenhouse gases distinguishes it from what is expected, for example, for a warming due to an increase in solar radiation.

Doug S
December 24, 2010 8:51 am

Love the graphics Willis, nice job trying to simplify a complex subject for a guy like me. I was wondering about the significance of gravity as I looked at your diagram. If the gravitational field strength on the earth varies over time, even just a little, would that introduce a significant delta in the energy balance? My first guess would be that a weaker G force would allow the water molecules to rise faster and farther up to TOA and release more energy to space. Conversely, a stronger G force would contain the water molecules closer to the earths surface and less energy would be released to TOA. Happy holidays to you and all the WUWT community.

December 24, 2010 9:02 am

Willis Eschenbach says:
December 24, 2010 at 1:37 am
(In a response to Keith Minto)
Keith, this is the path of a bit of energy. Certainly, other paths are taken. My point is that there is an energy path that completely dodges the surface. It comes in and just evaporates water. It does not raise the surface temperature at all. This gives a climate sensitivity of zero.

A simple question:
Are we measuring the energy in to the earth’s system at the same place that we measure the energy out (hopefully well above the atmosphere) and are we then measuring the energy reaching the earth’s surface?
Are we also measuring the reverse: energy leaving the earth’s surface and then the energy leaving the earth’s system?
If so, I would think we can easily observe what is happening although not necessarily exactly where or how.
Just wondering.

E.M.Smith
Editor
December 24, 2010 9:03 am

@Pamela Grey:
Folks need to dig in the dirt more often. It focuses you on the truth. I had cool garden soil later in the year than usual, but not too bad. You reminded me: decent worms, too; though they were closer to the surface than usual. We have it “the other way around” down here. They retreat deeper in summer as the soil dries and becomes too hot, rise when it’s cooler and damper. Hmmm… A “worm index” of temperature …. they only like a particular warmth / wetness band. So my worms also said cooler and wetter summer / fall.
At any rate, thanks for the insights from up your way. It’s folks that dig in the ground, see things, and have to grow food who know what’s happening. Guys sitting in office buildings in New York City playing Climate Nintendo don’t…
Joel Shore says: Follow your reasoning to its logical conclusion: In order to send more heat back into space, the upper troposphere must warm,
Your argument is based on the falacy of over averaging the data. In time and in space.
The AREA of warmth could increase a bit. The TIME during a given day that is warm could lengthen. The PEAK could rise briefy mid-day but otherwise have no long term effect.
The air THICKNESS could become less (as it has) allowing for different lapse rate effects.
And finally, “warmer” does not allow for the fact that the temperature is a POOR PROXY for HEAT. Expansion of the air as it rises (into our thinner air blanket) changes the temperature without a change of heat. So using words like “warmer” is misleading.
In this posting:
http://chiefio.wordpress.com/2010/12/02/does-convection-dominate/
I reference this paper:
http://hal.archives-ouvertes.fr/docs/00/31/68/93/PDF/angeo-19-1001-2001.pdf
that looks at the change of temperatures in the troposphere on a daily basis and finds that it tracks the daily cycle of the sun rising and setting and is different from place to place. So, with a 4th power function on radiation, and with the peak dumping the heat “right quick” inside HOURS… Just how does any “increased warming” of a small percent hang around more than a few minutes?
And with it dumped where it arrives, how does it warm “globally”?
So, yeah, I guess you are right. The top of a thunderstorm might be a nearly trivial degree warmer for a few seconds in the middle of the day, then return to the same stability point it was at before (thank you 4th power function…)
And this matters to me not one whit and has NO impact on the world.

Dave F
December 24, 2010 9:19 am

I wonder how many joules of energy were used to evaporate the water that was just dumped on Cali. And how much of that left to space.

Jim D
December 24, 2010 10:02 am

As I mentioned on the previous thread, and some have alluded to here, a missing piece in your equating dU to dT/S is time. If you add dU instantly, dT does not change instantly. In the earth system, if dU is added on one time scale and dT responds on a slower time scale, the value of S will change with time from a very small value to an asymptotic value maybe near 0.7 or 0.8 K per W/m2 over many years depending on what time window you are looking at. The climate sensitivity is the limit of what dT achieves given long enough to respond to dU. Clearly one year is not long enough and S is effectively smaller if you want to write the equation this way without time-dependence. You would improve the understanding a lot by using time dependence and a heat capacity (or thermal inertia) to relate energy to temperature. The equation for a forced harmonic oscillator, as I mentioned on the previous thread, is more appropriate, because it shows why higher frequencies have lower responses.

DocMartyn
December 24, 2010 10:45 am

We only have to think of one vector, up against the gravity well. We have a body of salt water and on its surface we have free, hydrogen bonded water molecules and water molecules that form the hydration sphere around ions. The very surface layer is dynamic and has less water molecules with binding affinity than those below as they are at an 3 to 2 dimensional interface. These molecules are being bombarded by gas molecules, mostly N2 and O2 which are about 60% heavier that a water molecule. The water molecules thus gain energy by being bombarded by the air and those on the right hand side of the Boltzmann distribution are converted into the gaseous state. The air ‘cherry-picks’ the water molecules with the highest energy content and the bulk aqueous phase cools.
You can direct a hair dryer, 1500W with a temperature of 65 degrees centigrade, across the surface of bowl of water and you will not get equilibrium. The water will never reach the temperature of the air stream; it will instead remain cool and evaporate. Pools of water are never at the same temperature as the noon air temperature, they are cooler. Air currents cool water by asset-stripping the hottest molecules, a similar system is used in super-cooling liquid He; laser light is fired over the surface and the gaseous He molecules, which have a higher than average temperature than the bulk, are energized and removed from the system in the bulk vacuum.
A gaseous water molecule is then carried aloft, always, on average, getting a better exchange of collision energy from N2 and O2 water molecules. They gain energy and move up the gravity well, again overall, the light water molecules cool the N2/O2 and move ever higher. As they go higher there are less and less collisions, as there are fewer molecules with the energy required to fight the gravity well. As they bounce up, they also slow, due to gravity. High up, a larger fraction of the molecules they encounter are water molecules, rather than N2/O2, due to gravity fractionation. The concentration of gas, at fixed temperature (or heat) with altitude is an exponential function that is dependent on its mass. Very light gasses like H2 and He are fractionated all the way into space and escape the gravity well completely whereas Cl2, which is fatally toxic at concentrations as low as 10 ppm, is harmless if one is above 12 feet . During WWI the rats in the trench’s were not equipped with gas masks and were able to survive German Cl2 attacks by climbing the few trees on no-mans-land.
Our water molecules are now high up and upon collision with each other can form hydrogen bonds. This can only happen to the molecules that are at the left hand side of the Boltzmann distribution; the remaining population is now hotter than the previous average. As the ice particle grows it gain mass, gaining mass makes further mass gain more likely. The collision energy between a gaseous water molecule and a, say (H2O)60, ice particle results in energy transfer to the particle and the binding of the incoming gaseous water molecule.
As the ice particle grows, it will begin to fall. The bottom face will be ablated from impacts with the atmosphere, absorbing heat, and the ice will melt. Upon melting it will maintain a temperature of freezing point, with the molecules on the right hand side of the Boltzmann distribution being stripped by collisions. In this super cooled form it is at the temperature where it can dissolve the largest amount of CO2. During its decent, the rain drop will collect CO2. 3.34 mgs of CO2 will be dissolved in every ml of water; 76 mM CO2 in pure, ice cold rain drops.
The bulk of this CO2 will enter the Seas and Oceans, which would explain the very rapid equilibrium between atmospheric and Oceanial CO2 sinks, with a t1/2 of about a decade.
The hotter it gets, the higher up water goes and the longer it falls.

Terry
December 24, 2010 11:08 am

Willis
As I understand it, the ocean HC is determined based on an assumption that it is only the top few hundred meters or so that contain the absorbed heat. Now any errors in this depth assumption will cause an energy imbalance, especially if there is significant mixing occurring at depth.
This is a topic that Pielke Snr had quite a “heated” discussion on with one of the AGW scientists earlier this year from memory.

December 24, 2010 11:32 am

Is a “tiny bit” of energy smaller or bigger than a bread box?

Stephen Wilde
December 24, 2010 12:48 pm

“But this does not keep it from warming the ocean, because of the turbulence of the top layer. So while evaporation is greatly enhanced as you say, some of the heat is mixed downwards, because of the mixing action of wind, waves, currents, and spray.”
Maybe so but where is the evidence ? The mixing actions themselves increase evaporation even without any more downward IR.
Evaporation is a net cooling process. It always takes more energy from the surrounding environment than is required to provoke it so in fact it accelerates energy flow out of the ocean faster than energy flows up from below. Hence the cooler layer at the top of the oceans that is 0.3C cooler than the ocean bulk below.
More evaporation from extra downward IR would either enhance the coolness of that topmost layer or be neutral. It certainly couldn’t weaken it by warming the water otherwise that cooler layer could not exist in the first place.
I’ve put that issue to a lot of AGW proponents over the years. No satisfactory evidence ever seen as regards a net warming effect within the oceans from more downward IR.

Baa Humbug
December 24, 2010 1:05 pm

Willis Eschenbach says:
December 24, 2010 at 12:36 pm
Willis I wouldn’t have thought too much of the LW warming at the surface skin would mix down. Remembering that the area with the warmest air, the tropics, are dominated by the so called “dolldrums”. Very little mixing happens in these areas.
Also, the water temperature in these areas is already quite high, 28-30DegC so that extra little bit of warming would increase evaporation quicker than any mixing might send it below. But as usual, I could be wrong 🙂
Merry Christmas mate, thankyou for taking the time to post all those wonderful thought provoking essays, I love ’em.
p.s. there is a way to get the likes of Lacis et al to respond to your posts. Post under a pseudonym or get a trusted friend to post for you 🙂 As soon as they see your name, they know they can’t con you with their confusing pseudo answers so they run.

Terry
December 24, 2010 1:37 pm

Willis
One other point re the assumptions that are made re the warmed ocean layer, ENSO and NAO surely ensure that a constant depth is not a valid assumption. Im still with Pielke on this in that there are major errors in the warmed layer calc.

Pamela Gray
December 24, 2010 1:39 pm

Re:
Steven Keohane says:
CO2 causes cooling?! WUWT?
Joel Shore responds saying:
“Yes…The increase in greenhouse gases is expected to cause cooling of the upper parts of the atmosphere (the stratosphere and apparently the mesosphere and thermosphere too). The different temperature change structure expected in the vertical from greenhouse gases distinguishes it from what is expected, for example, for a warming due to an increase in solar radiation.”
My response:
And this is connected to catastrophic AGW/change/disruption how? CO2 is a bit player in warming or cooling of the stratosphere and any temperature change at that level leaves us untouched by that change.
At issue is whether or not a fraction of a % increase in CO2 leads to substantially warmer/cooler/extremes here on terra firma, in the pressure systems that surround us, and in our oceans.
I give you this: if the heat you say should be here warming us up is instead escaping to space, then it should show up in the upper troposphere somewhere as it passes through. But we can’t find it there. That is not to say it isn’t there. It could be a problem with how we measure it. I think as heat is escaping, it is difficult to capture its signature in the upper troposphere as I think it is a very localized, sudden, transient phenomenon, with at least two different kinds of escape hatch that move around. I know through experience that localized strong radiative cooling is a sudden large drop in temperature, as if it were on an oiled slide. And that cooling that occurs in and around thunderstorms can take your breath away and leaves you searching for a jacket in the middle of summer. These localized great escapes of heat are quite different than the slow rise in temperatures in regional areas. Might this balance out?

Jim D
December 24, 2010 1:52 pm

Willis, there are only three ways to increase the evaporation rate, 1) the ocean gets warmer, 2) the wind gets stronger, 3) the air gets drier. Which of these are you proposing happens?
The point of my previous message was that if you had S correct, you would not need an extra term. S critically depends on the time over which you measure dT.

Pamela Gray
December 24, 2010 2:07 pm

Anybody have the full text to this:
http://adsabs.harvard.edu/abs/2010AGUFMSA31B1737L
It would seem that temperatures in the stratosphere and mesosphere, and tropopause are teleconnected to the solar wind. So change in the solar wind is hypothesized to have an affect on the measured temperatures at these layers. Mind you, I don’t see how these changes greatly affect whether or not we peel off clothes or add downy jackets to our daily wear.

Stephen Wilde
December 24, 2010 2:12 pm

Pamela,
Due to the little matter of the lapse rate the temperature of the stratosphere controls the height of the tropopause and thus the pressure distribution in the troposphere.
Anything that affects stratospheric temperatures affects climate.
It is conceded by established climatology that extra UV when the sun is more active warms the equatorial stratosphere to lower the tropopause there, widen the tropical air masses and push the jets poleward.
However none of the models can get that effect to account for the sheer scale of observed jetstream changes.
Consequently it is necessary to go a step further. The equatorial effect on stratospheric ozone is only a part of what goes on.
Additionally when the sun is active there is increased ozone destruction from around the stratopause upwards. That cools the upper layers (as actually observed) and additionally cools the stratosphere (also as observed) despite that equatorial UV effect.
That is the only theory I have ever seen proposed that actually fits what happened in the real world without resorting to the assumed effects of CO2 and CFCs.
The first step in proving that scenario has been the recent finding that between 2004 and 2007 ozone amounts increased above 45km for a warming effect despite the sun being quiet and presumably when the sun was active ozone above 45km declined for a cooling effect exactly as I propose.
So the idea that all we see is simply internal system variability is not tenable.
Furthermore anyone who does not accept my alternative explanation whilst failing to provide yet another alternative is impliedly accepting AGW theory on the basis that there is no other explanation (than AGW) for a cooling stratosphere at a time of active sun.
Anyone who disagrees with AGW theory must have an alternative natural explanation for the apparently anomalous temperature trend in the stratosphere when the sun was more active.
I have provided one which is consistent with more recent findings so if AGW is to be challenged successfully we have to run with it unless someone has a better idea.

Stephen Wilde
December 24, 2010 2:43 pm

“Jim D says:
December 24, 2010 at 1:52 pm
Willis, there are only three ways to increase the evaporation rate, 1) the ocean gets warmer, 2) the wind gets stronger, 3) the air gets drier”
1) is based on a false premise. There is no need for the ‘ocean’ to get warmer. All that is necessary is for additional energy from any source to be captured by individual water molecules that are already on the cusp of evaporating. The more energy that is supplied the more molecules are affected.
The additional energy brings forward the moment of evaporation and when evaporation occurs more energy is drawn from the environment than was required to provoke it. Evaporation is always a net cooling process.

harrywr2
December 24, 2010 2:59 pm

Jim D says:
“there are only three ways to increase the evaporation rate, 1) the ocean gets warmer, 2) the wind gets stronger, 3) the air gets drier. Which of these are you proposing happens?”
All three, it’s how a Hadley Cell works. If you simply accelerate Hadley Cells a little bit the excess heat ends up being dumped into the stratosphere.
http://www.newmediastudio.org/DataDiscovery/Hurr_ED_Center/Easterly_Waves/Trade_Winds/Trade_Winds_fig02.jpg

December 24, 2010 3:22 pm

What is the ratio of LWIR energy emitted from:
a) LWIR energy emitted from 1 sq Meter of the earth’s surface (assume ‘standard dirt’ LWIR emissivity values for the earth’s surface for this value)
to
b) LWIR energy emitted from a 1 sq Meter ‘column’ of air running from just above the surface of the earth to the TOA and this would be LWIR emited upwards only from that atmospheric column (one could take value of overall LWIR exiting TOA and subtract the amount computed coming from the surface a) above to derive _that_ which is due to LWIR emissions from the atmosphere alone).
I will venture to say the value from the surface >> than that of that atmospheric column of air (LWIR out to space due to LWIR from air molecules comprising a column of air from the surface of earth to TOA altitude) .
.

Pamela Gray
December 24, 2010 4:47 pm

Please Steven, let’s call that a hypothesis. It is not anywhere near being a theory. And when an idea is a hypothesis, the null hypothesis rules. Your posts, illuminating a hypothesis, should start with a question, which is the proper way to talk about a hypothesis. A theory is a statement.

Pamela Gray
December 24, 2010 4:50 pm

Sorry about your name. Replace “Steven” with Stephen please. Again I apologize. However, your post is so filled with assumptions and either/or statements that it has my dander up and my red hair flaming!

Schrodinger's Cat
December 24, 2010 5:26 pm

I refer to the longwave (IR) causing evaporation from the sea. I agree with the comments of Willis and Stephen Wilde.
Thinking about this in more depth, the stretching, bending, wagging and rotational effects are IR frequency dependent and characterisitic of the water molecule infra red signature. So more IR photons will cause more energetic vibrations of water molecules. The energy of the molecules can be regarded as thermal or kinetic. Possessing a higher energy means that these molecules behave in the same way as if they were heated. If you like, they were just heated efficiently by giving them energy at wavelengths at which they absorbed.
In addition to the enhanced movement of the hydrogen-oxygen bonds, any further energy input will cause the water molecules to move about more vigorously and break away from the surface into the atmosphere. This involves the effect of vapour pressure which itself is a function of temperature.
Then we get the latent heat of evaporation, which is very large.
I’m enjoying this debate but more pressing things are happening. I’m typing this after returning from a Christmas Eve party. All the family have gone to bed and I’ve just realised that my wife has removed the bottle of wine that was to the left of my keyboard. I am now into Christmas day by more than one hour so it would be very Geekish (is that a word?) to continue.
Very best wishes to Anthony and family, the moderators, Willis, and all of you.
[The mods strongly recommend finding (1) your wife or (b) your bottle of wine.
Your choice of order will be important in determining what you find after either (2) or (a) ….. 8<) Robt]

Jim D
December 24, 2010 5:26 pm

Stephen Wilde, there is an equilibrium vapor amount at a water-air interface given by Clausius-Clapeyron. There is no way to just add to the evaporation rate if temperature there is not changing. It is incorrect to think IR photons just cause evaporation, and not heating of water molecules. How can they distinguish, and does a single photon even have enough energy to evaporate a molecule by supplying sufficient latent heat? Not quite.

December 24, 2010 5:33 pm

Pamela Gray says:
“Please Steven, let’s call that a hypothesis. It is not anywhere near being a theory. And when an idea is a hypothesis, the null hypothesis rules. Your posts, illuminating a hypothesis, should start with a question, which is the proper way to talk about a hypothesis. A theory is a statement.”
Excellent post, Pamela. Concise. There are way too many assumptions floating around.
And a great post by Willis, as usual.

JimF
December 24, 2010 5:46 pm

Willis: As many times before, BRAVO. I’m certain you’ve hit on the extraordinarily large-scale phenomenon that obviates the otherwise static CO2 begets H2O begets even more H2O, all the while broasting earth with LW radiation of the AGW clique.
Here, the H2O just bolts into the stratosphere, gives back its heat as it condenses out to the (now) dry air that goes on to rid the heat to space (and form deserts where it comes back to earth on cooling, plus form the trade winds, yawn!). The water now falls back to earth cooling it, and maybe, sucks up a bunch of CO2 in doing so, putting that varmint in the confines of the ocean.
And, following what Pamela Gray says:
December 24, 2010 at 7:36 am
“…I can’t see how this tiny bit more heat (anthropogenic) at the surface will have much of an affect on land….”
the same thing goes on at temperate latitudes during summer, as massive thunderstorm systems set up in places like Oklahoma and Wisconsin and the Ukraine and so on.
So we now have a system that simply cuts across the radiation in/radiation out/radiation back models, and that incidentally, results in major features of the earth’s surface (deserts) and atmosphere (winds). My guess is that this is one of the key elements of the earth’s climate. All because of one little, uniquely odd molecule called WATER. There ain’t nuthin’ like it anywhere else that we know of.
Keep up the good work.

Jim D
December 24, 2010 6:05 pm

Schrodingers Cat, you are starting to come around to the idea that IR can heat the surface, and that in turn gives more evaporation. You don’t shortcut the heating like the others here seem to be trying to do.
The net effect of IR is actually cooling, because there is a net loss of longwave energy from the water surface, but its cooling effect is slightly offset by adding CO2 the way adding cloud cover does relative to a clear night. It is easy to imagine that an ocean under cloud cover at night is on average going to be warmer than one under clear skies. Same thing with CO2. No one talks about cloud cover enhancing evaporation, but it is the same physics as far as photons are concerned.

JFD
December 24, 2010 7:52 pm

Willis, I just found this topic and am leaving on vacation early in the morning so must make this short and sweet. I will carefully read your treatise and all comments when I get back in a week.
My take is that the “missing energy” comes from the production of groundwater from slow to recharge aquifers. This water is not in equilibrium with the water present on the surface of the earth. The largest, by far, bulk of the produced water is used for irrigation of food and fodder vegetation. In 1995, this fossil production amounted to 830 cubic kilometers.
The produced liquid water undergoes evapotranspiration and discharges into the air as water vapor, thereby changing the potential energy of the liquid water into kinetic energy at constant temperature. The water vapor rises until it condenses changing the kinetic energy back into potential energy there by releasing the absorbed latent heat as specific heat, which warms the air. After that the condensed liquid water causes the oceans to rise a calculated 2.6 mm per year. At that point the groundwater is in equilibrium with the surface water and becomes part of the hydrological cycle. So the temperature increase and the ocean level increase are a function of only they yearly produced water volume.
The 830 cubic kilometers of groundwater per year releases a huge amount of energy into the atmosphere that was not there before. In recent years, due to dropping aquifer levels, the volume of groundwater pumped for irrigation has started to decline, especially in US and parts of China. This groundwater from slow or no recharge aquifers fits your “model” very well, I think.
Additionally, I believe that there is a functional temperature control valve in or very near the Tropopause that thermodynamically displaces a part of the 4 ppm water vapor as the carbon dioxide content increases due to partial pressure effects. I base this temperature control scenario on the fact that the NOAA measured humidities in the mid and upper Troposphere have decreased each year since 1958.
I can go into more detail but that is the essence of my studies on the causation of global warming and ocean level rise.
JFD

DanDaly
December 24, 2010 7:59 pm

Not sure where this leads but: When they made me take meteorology to get my journalism degree, the instructor said that as water evoporates, it absorbs heat and, typically, rises; however, when water condenses at altitude, typically, droplets fall and, by virtue of friction, generate heat, which is imparted to the surrounding air. Thus, evaporation and condensation both absorb and generate heat. The heat appears to be moved, as you say, from the surface to … somewhere else. While water vaper may tranfer heat to space at altitude, when water condenses it and the surrounding atmosphere is heated by friction when it falls. If I recall the lessons, surface temperature declines by about the same increase in temperature cause by friction in the upper atmostphere.
Not that this makes a big difference, but there does seem to be a bunch of entropy floating around.

Joel Shore
December 24, 2010 8:45 pm

Dan Daly: You don’t need to take friction into account to explain why the atmosphere is heated when water condenses. It happens for the same reason why water must absorb heat to vaporize: a phase change from the liquid to the gas requires the absorption of energy and a phase change from gas to liquid requires the release of energy. It is called the “latent heat of vaporization”.
Willis Eschenbach says:

This is one of the times we seem to be mis-communicating. I’m not talking about models vs. reality. I’m talking about observations versus the current climate theoretical paradigm.
Current climate theory says that ∆U (energy leaving the system) = ∆T (surface temperature change) / S (climate sensitivity).
As the example above shows, that’s not true. And climate models and their antics, while interesting, don’t even enter the discussion.

I think then that you have created and destroyed a strawman. I don’t think climate scientists would claim that climate sensitivity applies down to these time- and length-scales. Climate sensitivity is measured by what happens over the long run, not about a tiny fluctuation. After all, over the short term there is no doubt that energy can go into a lot of different places…It can be absorbed into or liberated from the ocean, and so forth.
And, if you think the effect that you describe is a significant effect over the long term, then you should work out the consequences of your ideas and what they imply, which is where I was going with your notion here.

My best holiday wishes to you,
w.

Happy holidays and best wishes to you as well!

Joel Shore
December 24, 2010 8:56 pm

E.M. Smith says:

Your argument is based on the falacy of over averaging the data. In time and in space.
The AREA of warmth could increase a bit. The TIME during a given day that is warm could lengthen. The PEAK could rise briefy mid-day but otherwise have no long term effect.
The air THICKNESS could become less (as it has) allowing for different lapse rate effects.
And finally, “warmer” does not allow for the fact that the temperature is a POOR PROXY for HEAT. Expansion of the air as it rises (into our thinner air blanket) changes the temperature without a change of heat. So using words like “warmer” is misleading.
In this posting:
http://chiefio.wordpress.com/2010/12/02/does-convection-dominate/
I reference this paper:
http://hal.archives-ouvertes.fr/docs/00/31/68/93/PDF/angeo-19-1001-2001.pdf
that looks at the change of temperatures in the troposphere on a daily basis and finds that it tracks the daily cycle of the sun rising and setting and is different from place to place. So, with a 4th power function on radiation, and with the peak dumping the heat “right quick” inside HOURS… Just how does any “increased warming” of a small percent hang around more than a few minutes?
And with it dumped where it arrives, how does it warm “globally”?
So, yeah, I guess you are right. The top of a thunderstorm might be a nearly trivial degree warmer for a few seconds in the middle of the day, then return to the same stability point it was at before (thank you 4th power function…)
And this matters to me not one whit and has NO impact on the world.

Well, frankly, this sounds like a lot of hand-waving to me. I don’t think you’ll find that the 4th power function is all that dramatic over the absolute temperature ranges that we are talking about. (Over a small enough temperature range, the T^4 will look linear anyway, just with a slope four times as great as a T^1. So, as an example, if the absolute temperature increases by 1%, then the radiation increases by 4.06%. If the absolute temperature increase by 10% instead, then the radiation increases by 46.4%.) If we were talking about absolute temperatures that increased by factors of 2 or 3 or something, then yeah, the nonlinearity of the T^4 power would be quite important; however, for changes of 1 or 5 or even 10%, it ain’t very.

Keith Minto
December 24, 2010 10:36 pm

NovaReason says:
December 24, 2010 at 12:34 am
Thanks, that it it in a nutshell. It is just a thought experiment as I have not seen this area discussed before. Yes it could generated heat lower down, the above mentioned NS article says 5% of the condensation energy could be radiative and if the nuclei were continuously generated then the heat would rise together with the extraordinary buoyancy of water vapour. No net heat gain, just a spread of heat generation points closer to sea level. I will keep searching.
Christmas dinner calls and after all this is Willis’ thread.

Stephen Wilde
December 24, 2010 11:19 pm

Jim D said:
” It is easy to imagine that an ocean under cloud cover at night is on average going to be warmer than one under clear skies. Same thing with CO2. No one talks about cloud cover enhancing evaporation, but it is the same physics as far as photons are concerned.”
Jim, are you sure ?
Isn’t it just the air above the water that is made warmer by the cloud ?
The cloud prevents the energy in the air from escaping upwards as quickly as it otherwise would have done but at the same time it stops solar shortwave energy from penetrating the water. So which effect prevails over a diurnal cycle, the slight reduction in evaporation rates or the cooling effect of increased albedo ?
Generally speaking a cloudier world is a cooler world is it not ?
And the cloud analogy is not really applicable because it operates by alteriung optical depth but according to Miskolczi the increased CO2 has failed to alter global optical depth for the past 60 years which is the period of time when human effects are supposed to have affected natural processes.

Stephen Wilde
December 24, 2010 11:25 pm

Jim D said:
“It is incorrect to think IR photons just cause evaporation, and not heating of water molecules. How can they distinguish, and does a single photon even have enough energy to evaporate a molecule by supplying sufficient latent heat? Not quite.”
I said that water molecules are heated but that the timing of evaporation is then accelerated so that there is no net heating of the ocean bulk.
A single photon does not supply latent heat. it supplies sensible heat which then provokes earlier evaporation and when evaporation occurs more energy is drawn from the local environment as latent heat than was acquired from the additional photon(s) that directly caused the change of state.

Stephen Wilde
December 24, 2010 11:33 pm

“Pamela Gray says:
December 24, 2010 at 4:47 pm
Please Steven, let’s call that a hypothesis. It is not anywhere near being a theory. And when an idea is a hypothesis, the null hypothesis rules. Your posts, illuminating a hypothesis, should start with a question, which is the proper way to talk about a hypothesis. A theory is a statement.”
It would be more helpful if you could point out what is wrong with my theory/hypothesis rather than just criticising my non scientist writing style.
After all it does fit the observations rather well whereas established theories (or are they just hypotheses due to all their underlying assumptions) are just generating confusion because they do not fit observations without invoking an assumed effect from anthropogenic CO2 and CFcs.
How can one oppose AGW theory without having an alternative explanation as to why the stratosphere cooled whilst the sun was more active in the late 20th century ?
Without a ‘natura” explanation their assumptions become the only game in town don’t they ?

DocMartyn
December 25, 2010 5:08 am

Jim D said:
“It is incorrect to think IR photons just cause evaporation, and not heating of water molecules. How can they distinguish, and does a single photon even have enough energy to evaporate a molecule by supplying sufficient latent heat? Not quite.”
In the 3D bulk phase each water molecule has four strong hydrogen bonds, with to -H donors and two (:) receivers. At the atmospheric interface, water has a 2D surface, the top layer will have on average only 2 hydrogen bonds. As well as being bombarded by IR from the atmosphere, these molecules will also be bombarded by IR from below; from the thermal energy of the bulk water.
A little experiment for the summer. Take two children’s paddling pools and fill with the same amount of water and place a thermometer in each. Hang a thermometer on a thread above each. Pour a liter of light machine oil on one to stop evaporation. Place in noon-day sun. The water temperature of the oiled pool will be higher than the other, but the surface, air temperature will be lower.

Pamela Gray
December 25, 2010 5:40 am

Stephen, your statement, “Anything that affects stratospheric temperatures affects climate.” is an hypothesis without merit.
But first, my opinion: It is hypothesized that trace amounts of greenhouse gases in the stratosphere do have an affect on temperatures there (depending on the altitude). This can be calculated and demonstrated to match observations. But ozone changes have the larger affect, also calculated and demonstrated to match observations. The caveat, the temperatures of the stratosphere have only been measured for at best 2 decades, maybe 3. Anything prior to that is just a calculation based on hypothesized mechanisms. Therefore, all papers postulating this or that driver of stratospheric temperature changes are, in my opinion hypotheses that have yet to see the light of theory. Some have more merit than others.
Now to your statement. What is more established is that stratospheric temperature teleconnections with driving potential to lower tropospheric temperature patterns have no mathematical constructs. Not enough energy to be a driver down here. Metaphorically, to stir a pudding, you have to have something stronger than a feather. You state that jet stream movement requires quite a bit of energy. Your hypothesis has not been calculated yet to my knowledge here on this blog.
If you present yourself as an amateur scientist, that does not mean that you can function well below what is expected of professional scientists without being called on the carpet.
You postulate an unproven hypothesis, nothing more, and not very well in my opinion.

Jim D
December 25, 2010 7:42 am

Stephen, actually on clear nights the ground cools more than the air, being a black body. On cloudy nights it cools less, same for the ocean. CO2 has the warming effect of clouds, though much weaker and sustained, while it doesn’t have the cooling effect in the day due to being transparent to light. If you imagine the ocean subjected to such forcing it can’t help but get warmer.
Hopefully you don’t really believe in Miskolczi and his radiosondes. It goes against basic physical science in textbooks (not just climate science). It will go the way of cold fusion soon enough when no one else finds what he did with his selected data.
On the other message, yes photons heat water molecules. Why can’t they just distribute that heat rather than evaporating. This is where the whole idea of evaporating balancing the GHG falls apart. Evaporation does increase, but only after the surface has warmed to a new Clausius-Clapeyron equilibrium. This is the basis of the water vapor feedback. You are saying evaporation cools it back to the original temperature. No, because the photons are still there keeping it warmer. The new equilibrium with these photons is a warmer temperature with more vapor at the interface.
DocMartyn, the IR photons don’t have enough energy to supply the latent heat needed to evaporate water. They will therefore all just go into heating the water. The evaporation rate is governed by the sustained interface equilibrium between vapor and water, which depends on temperature.

DocMartyn
December 25, 2010 11:46 am

“DocMartyn, the IR photons don’t have enough energy to supply the latent heat needed to evaporate water. They will therefore all just go into heating the water. The evaporation rate is governed by the sustained interface equilibrium between vapor and water, which depends on temperature.”
Have a look at the actual structures of water molecules in the liquid phase. The reason that water and ammonia have such high boiling points is due to hydrogen bonding. The water molecules at a water:air interface do not have the same hydrogen bonding network as those beneath. Moreover, as energy is the thing that puts a molecule there, the water molecules at the surface will have higher than average energy. There is a big difference between boiling water and evaporating it under a gas stream. Chemists use N2/vacuum lines to ‘dry’ salts without heating. You can get rid of water while dropping the temperature. If you play a nitrogen stream at ambient temperature over a salt solution you dry it and drop its temperature. The gas stream strips water molecules, preferentially stripping molecules with the highest energy.

Joe Lalonde
December 25, 2010 12:58 pm

Willis,
Something NOT considered is the pattern of WIND.
Wind has a rotational pattern.
How can I prove this?
A snowflake! When evaporating water vapour is freezing with a wind, the water vapour has to be spinning two dimensionally in order to achieve a flat plane of the snowflake.

JP Miller
December 25, 2010 4:15 pm

Pamela, judgmental criticism by itself is too easy; more difficult is to take a partially/ poorly-formed idea with a grain of potential and improve it. Constructive criticism requires suggesting improvements. You obviously have much to offer. Do you think your expertise is worth offering to Stephen’s thinking? In other words, could he “have something there” or not? If “no,” it would be better if you would cut to the chase regarding why not, while cutting out the unhelpful sniping. If “yes,” then it would be great if you could help him develop it.

December 25, 2010 6:14 pm

Interesting discussion. Living in the Keys I tend to think more about hurricanes than snow. The average hurricane losses about 600 trillion Watts per day in rain out. Wind energy is about 1 trillion Watts per day. Blocked incoming radiation is probably between 1 and 10 trillion Watts per day. Wave energy produced I have never seen estimated. I just think it is amazing how big that tiny bit of energy can be for just one storm day.

David
December 26, 2010 1:40 am

Willis, is it not possible that some systems receive energy quickly, but manifest it (meaning make it perceptible to us) slowly? Do we really have the understanding and sensitivity in all of our measuring to capture the energy budget as it changes form, phase and location, or are there possibly slow changes in thermocline depths, hydrologic cycle speeds, atmospheric elevations, Jet stream latitudes, cloud formations and locations, sea life blooms and crashes, large ocean currents etc, that can receive energy quickly but manifest it as temperature slowly or even imperceptibly in regard to our ability to capture these changes?
Each wavelength of incoming TSI has a different residence time within the atmosphere, land and ocean. This residence time is of course affected by it own inherent properties as well as all of the material it encounters. Only two things can effect the energy content of any system in a radiative balance. Either a change in the input, or a change in the “residence time” of some aspect of those energies within the system.” The longer the “residence time” the greater the energy sink capacity. The greater the energy capacity, the longer it takes for any change to manifest, and in the case of OHC this involves years, not annually. This makes your caption statement for Figure two inaccurate; “ Figure 2. The missing energy puzzle. Every year, the amount of energy entering the system (red) should equal the energy leaving the system (light blue) plus the energy going into/out of the ocean (dark blue). It doesn’t.”
According to John Daly’s references many ocean inputs take about 8 years to manifest in the atmosphere. http://tallbloke.wordpress.com/2010/12/06/john-l-daly-the-deep-blue-sea/
Evaporation conduction of latent heat may vary far more then realized and set a limit on further tropical temperature increases. (Newell & Dopplick’s (1979) calculations that tropical temperatures cannot rise any further.)
Also have you looked at the bi-annual cycles in all that you are measuring to see if the earth’s seasonal energy pulse can reveal some of this mystery? Sunlight, falling on the Earth when it’s about 3,000,000 miles closer to the sun in January, is about 7% more intense than in July. Because the Northern Hemisphere has more land which heats easier then water most people state that the Earth’s average temperature is about 4 degrees F higher in July than January, when in fact they should be stating that the ATMOSPHERE is 4 degrees higher in July. In January this extra SW energy is being pumped into the oceans where the “residence time” within the Earth’s ocean land and atmosphere is the longest
As these immense changes in SWR TSI happen bi-annually, then how much and how rapidly changes in those things you measured in figure 2 match these bi-annual changes should give deeper insight relative to heat and energy flux within our earth system. I think figure two needs to be refined to capture this seasonal flux.
Thank you for your excellent articles that both educate and stimulate further thought.

Stephen Wilde
December 26, 2010 2:21 am

Pamela Gray sais:
“Stephen, your statement, “Anything that affects stratospheric temperatures affects climate.” is an hypothesis without merit. ”
On the contrary, it is established climatology. I’m puzzled by your lack of awareness of that.
Even a temprary warming of the stratosphere from say a so called sudden stratospheric warming event changes the pressure distribution below to mimic a negative AO.
Also differential UV warming of the equatorial stratosphere from imteractions with ozone in the stratosphere is supposed to lower the tropopause, widen the tropical air masses and push the jets poleward.
I am not putting forward speculative statements. Instead I take established propositions, compare them with observations and set out some of the logical implications.
What puzzles me greatly is that since CO2 became the universal bete noir such an approach appears to have been neglected by the climate establishment yet it should be at the heart of the science.
Your personal opinion of the quality or otherwise of my work is of little interest. It has received favourable responses elsewhere.

Stephen Wilde
December 26, 2010 2:34 am

“Jim D said:
“It is incorrect to think IR photons just cause evaporation, and not heating of water molecules. How can they distinguish, and does a single photon even have enough energy to evaporate a molecule by supplying sufficient latent heat? Not quite.”
A single photon does not have enougfh energy to evaporate a molecule on its own. It simply adds to the energy already held by the molecule to bring forward the timing of evaoration. Evaporation is an ongoing processs always happening due to pressure, density and temperature differentials. IR just speeds it up rather than causing it ab initio.
The latent heat of evaporation is not provided by the incoming photon even if it tips the moleclue into evaporating. The latent heat is taken from the surrounding environment and is always far greater than the energy in the photon that tipped the molecule into a change of state.
So one photon with say one unit of energy can tip a molecule of water into changing state to a vapour but the surrounding environment loses many more units of energy to latent heat for a net cooling effect.
If you think there is any surpus energy left over from incoming photons of IR after such a net cooling process has taken place then you must prove it. As it stands your position and that of AGW theory in general is illogical.

Stephen Wilde
December 26, 2010 2:38 am

Jim D said:
“Hopefully you don’t really believe in Miskolczi and his radiosondes”
What has ‘belief’ got to do with anything ?
So has the atmospheric optical depth changed over the past 60 years ? I have seen lots of personal comments about him and his work but without that point being addressed.

Stephen Wilde
December 26, 2010 6:25 am

Jim D said:
“CO2 has the warming effect of clouds, though much weaker and sustained, while it doesn’t have the cooling effect in the day due to being transparent to light.”
CO2 does have the same ability as clouds to slow down the upward rate of energy loss. No dispute.
However CO2 is not perfectly transparent to light which is why I raised the optical depth issue. Co2 molecules block and reflect upwards again incoming energy just as much as they block and reflect downwards again any outgoing energy.
So just like clouds, but to a much lesser degree (as you point out) extra CO2 has an effect on both incoming and outgoing energy.
So to get a significant net warming effect from CO2 one needs also to see a change in optical depth.
If there is no change in optical depth then it must be assumed that extra downward IR from more CO2 is balanced by more upward IR from blocked incoming energy redirected back upward.
Just as for clouds the extra IR redirected back downward when the cloud is present is more than compensated for by downward energy being blocked but because clouds are much more opaque than CO2 then for clouds the amount of incoming energy sent back upward is much greater than the amount of outgoing energy bounced back downward. That is how clouds increase the optical depth. Perfect transparency can only achieve equality of effect. Even perfect transparency cannot make molecules send more back down than they send back up yet that is what is claimed for CO2 molecules.
In order to send more energy down than than is rejected and sent back up an individual molecule has to generate more energy internally like a sort of perpetual motion machine.
However increased total atmospheric density from more molecules overall or from an increase in denser molecules resulting in greater optical depth will send more energy back down than is rejected to space until a new and higher equilibrium temperature is reached because it is density that slows down the net rate of energy transmission. That is why planets covered by dense clouds of any material can get very hot at the surface.
Optical depth changes (equating to density) are the measure of the net balance whether it be clouds or GHGs or both. So has optical depth changed or hasn’t it ?

Jim D
December 26, 2010 7:52 am

Regarding Miskolczi, I have not seen anyone confirm his data, but on the contrary satellite data which is much more precise in directly measuring the effect of CO2, does show its effect increasing. For me the weight of direct measurements is greater than the weight of Miskolczi’s indirect radiosonde approach, but people can choose for themselves.

Jim D
December 26, 2010 8:03 am

Stephen Wilde said
“However increased total atmospheric density from more molecules overall or from an increase in denser molecules resulting in greater optical depth will send more energy back down than is rejected to space until a new and higher equilibrium temperature is reached because it is density that slows down the net rate of energy transmission. That is why planets covered by dense clouds of any material can get very hot at the surface.”
This is almost correct, except that optical depth depends not only on density but atmospheric constituents, especially the greenhouse gas percentages.
Also the idea that CO2 has any significant effect on visible light compared to its infra-red effect is not borne out by the facts.

kuhnkat
December 26, 2010 8:43 am

Stephen Wilde,
You obviously are much more knowledgeable than I, but, I would ask you to change your terminology on CO2. It absorbs and emits LW. There is a fundamental difference in what happens in these two scenarios. If CO2 reflected and scattered only it could not heat the surrounding air mass.

Stephen Wilde
December 26, 2010 9:36 am

Jim,
Can you link me to any satellite data relating to optical depth , please?
Atmospheric constituents affect optical depth too, agreed. It’s hard to cover everything in a blog post without losing one’s drift. However, absorbing solar LR and radiating it back upwards is similar to the reflection of visible wavelengths by clouds.
kuhnkat,
Noted, thank you. However there is some reflection and scattering too is there not ? And upward or even sideways radiation of LR has much the same effect as reflection and scattering in overall energy transfer terms.

Joel Shore
December 26, 2010 10:18 am

Stephen Wilde says:

Jim,
Can you link me to any satellite data relating to optical depth , please?

There are a bunch of references given in here: http://skepticalscience.com/empirical-evidence-for-co2-enhanced-greenhouse-effect.htm
There are also studies relevant specifically to the water vapor feedback such as this paper by Soden et al. http://www.sciencemag.org/content/310/5749/841.abstract and various papers by Dessler.

Stephen Wilde
December 26, 2010 10:29 am

Interesting stuff about optical depth here:
http://icecap.us/images/uploads/WHY_CLIMATE_MODELS_FAIL.pdf
Note that water vapour varies much more that CO2. But instead of causing a huge positive feedback, water caused a strong negative feedback, and reduced the optical depth trend (green line) to 2.58 10-5 X 60 years = + 0.0015, or about 0.083% in 60 years. This means that water vapour has offset 78% of the greenhouse effect of CO2 change over the last 60 years. This is very significant! Note also that there has been a dramatic drop in water vapour at all altitudes in 2008, which is not included in the above calculations. I do recognize that the early NOAA data might be less accurate than more recent data, but this is what the data shows. This data supports Miskowski’s theory of the greenhouse effect. Miskolcki shows the standard theory uses inappropriate boundary conditions. When real boundary conditions are used, he shows that the atmosphere maintains a saturated greenhouse effect, controlled by water vapor content.

Stephen Wilde
December 26, 2010 11:28 am

Joel,
I don’t see anything there about optical depth over the last 60 years as measured by satellites. Have missed something ?

Jim D
December 26, 2010 11:28 am

I thank Joel for digging up those references on direct observations of CO2 affecting outgoing longwave. The Griggs reference is, I think, the particular one I had in my memory, or something very like it.
Regarding water vapor we have
http://www.ipcc.ch/publications_and_data/ar4/wg1/en/figure-3-20.html
from the IPCC report that uses satellite data and total column water vapor, which is more important than mid-tropospheric values, since most of it is near the surface.
On the other point, atmospheric constituents are vital to the effect you were talking about since mass is not varying. This alone explains the greenhouse effect, and how much CO2 can change it.

Stephen Wilde
December 26, 2010 11:33 am

Jim, Joel,
You need to explain to me why you are referring me to data about Outgoing Longwave Radiation when I am asking about optical depth.

Jim D
December 26, 2010 11:47 am

If you are reducing OLR, you are increasing optical depth as long as the surface temperature isn’t cooling to compensate.

Jim D
December 26, 2010 1:15 pm

Willis, we all know that when you make a step change in forcing dU, T doesn’t respond immediately. S is effectively very small or zero in the first seconds. How long does S take to reach the full climate sensitivity? I say several years at least. What does this mean? It means the “missing energy” is just a temporary situation representing thermal inertia, because changing the surface temperature in a climate system actually has deeper impacts in the ocean and soil that have to get into the new equilibrium. The missing surface energy is just going into warming these deeper layers. Averaging over long enough beyond the dU change, the missing energy goes to zero, as T reaches its full response.

Stephen Wilde
December 26, 2010 2:05 pm

“Jim D says:
December 26, 2010 at 11:47 am
If you are reducing OLR, you are increasing optical depth as long as the surface temperature isn’t cooling to compensate.”
I’d prefer to work from a direct measure of optical depth so as to exclude the effect of temperature variability at the surface or in the atmospheric column.
OLR changes could be a consequence of factors other than a change in optical depth. For example, a change in the vertical temperature profile could change OLR without changing optical depth.

Jim D
December 26, 2010 3:39 pm

I am still trying to understand Miskolczi’s definition of optical depth. It seems to be St/Su where St is the part in the window region of the OLR. If so, this is not including the greenhouse gas OLR in his definition, which therefore is not surprisingly insensitive to GHGs. Is this a correct interpretation? Why not use total OLR over surface emission which gives optical depths nearer 0.5 instead of 1.8. His high optical depth is because he is taking only the window fraction, which is just a function of the window width he chooses.

Pamela Gray
December 26, 2010 6:25 pm

Stephen, this paper is a good short article that reflects my current understanding of the teleconnections between the stratosphere and troposphere with regards to the AO. The bottom line is that most studies see the primary relationship as non-influential in terms of the stratosphere as a driver of the AO.
http://www-eaps.mit.edu/~rap/papers/AO_revised.pdf

Joel Shore
December 26, 2010 7:12 pm

Stephen Wilde says:

I do recognize that the early NOAA data might be less accurate than more recent data, but this is what the data shows.

Actually, that radiosonde humidity data is complete garbage for determining long-term trends, as has long been recognized in the literature (you can find a reference in the Soden paper that I linked to) and does not agree with satellite data. Note that the Soden paper shows that both the long-term trend and the shorter-term fluctuations (e.g., due to El Nino and La Nina) show the positive water vapor feedback.

I don’t see anything there about optical depth over the last 60 years as measured by satellites. Have missed something ?

We haven’t had satellites monitoring this stuff for 60 years. The Soden paper essentially measure relative humidity (by measuring the infrared radiation passing through the atmosphere in certain frequency bands), which is the same thing that the reference that you gives reports…except that your reference uses radiosonde data of this that is known to be unreliable, especially for assessing long-term trends.

Note also that there has been a dramatic drop in water vapour at all altitudes in 2008, which is not included in the above calculations.

…Which is exactly what one expects if water vapor is a positive feedback, given that the temperatures in 2008 dropped quite dramatically due to a strong La Nina.

Stephen Wilde
December 27, 2010 4:00 am

Pamela,
That link is limited to short term wind induced anomalies.
For climate purposes we need to consider long term stratospheric temperature trends such as the cooling observed throughout the late 20th century tropospheric warming trend. Conventional climatology says that all the layers should warm together when the sun is active but they didn’t. That is what encouraged the AGW proponents to propose human intervention.
Anyway, back to the point. A warmer stratosphere lowers the tropopause and a cooler stratosphere raises the tropopause whatever the cause of the temperature change.
The reason is the lapse rate so look that up if you need to.
The height of the tropopause dictates the size, intensity and position of the air circulation systems in the troposphere.
So if one changes stratospheric temperatures the perceived climate will change because the air circulation systems shift, especially the jets.
That is established climatology.

Stephen Wilde
December 27, 2010 4:05 am

Joel,
Actually you have commented on the points made in the extract that I quoted. I should have put it in quote marks.
So I am left having to sort out the mess caused by two ostensible experts (which I am not) who present entirely different assessments.
However the Miskolczi finding fits well with my proposition about a variable speed for the water cycle such that any increased optical depth would be negated by an increase in the speed of the water cycle.
So I’ll go with Miskolczi until I find something better.

Pamela Gray
December 27, 2010 5:49 am

I’m wondering if:
Since the heat they are looking for is in the form of LW radiation, it seems logical to ask where can such heat be stored? If not the oceans (an unlikely source because it can’t store any of it except a tiny fraction that may, as some have suggested, get mixed in at the top most layer along with short wave solar radiation and eventually make it to the deeper layer via the http://oceanmotion.org/html/impact/conveyor.htm), then where? Could it be that this is where the notion starts about weather pattern disruption? That this LW heat, that at one time could simply escape but now gets bounced back to the surface and can’t escape, is causing almost immediate changes to weather? Could this be where the idea of “extreme” comes from?
If this is part of AGWer thinking, it forms a conundrum. If it’s getting stored in deep layers of the oceans, it can’t be causing “climatic disruption” (their term for what I think is more properly called weather pattern disruption) at the surface. If its being used at the surface to cause weather pattern disruption, it can’t be getting stored somewhere. If my “wonderings” are full of it, enlighten me.

harrywr2
December 27, 2010 7:53 am

The models do a poor job with Hadley cells. The observed vs modeled changes are significantly different. Climate models that can’t reasonably simulate the observed changes in Hadley cells are pretty useless. Of course since we don’t know why the Hadley cells are acting the way they are it’s hard to simulate them.
http://journals.ametsoc.org/doi/abs/10.1175/2008JCLI2620.1
The authors find that observed widening cannot be explained by natural variability. This observed widening is also significantly larger than in simulations of the twentieth and twenty-first centuries. These results illustrate the need for further investigation into the discrepancy between the observed and simulated widening of the Hadley cell.

Joel Shore
December 27, 2010 8:43 am

Stephen Wilde says:

However the Miskolczi finding fits well with my proposition about a variable speed for the water cycle such that any increased optical depth would be negated by an increase in the speed of the water cycle.
So I’ll go with Miskolczi until I find something better.

Well, don’t be surprised to find yourself pretty lonely then in the scientific community, given that Miskolczi uses a data set that nobody seems to believe is accurate for the purposes that he uses it for.

Joel Shore
December 27, 2010 9:07 am

I am using annual data. I have never heard anyone claim that climate sensitivity varies from year to year. If it does, what is the nature of the variations? Does it change in regards to temperature or other variables? How much does it vary?
Because on an annual basis, what comes in has to equal what goes out plus what is stored.

But, I think Trenberth’s whole point is that we can’t measure all these quantities accurately enough now to see what is happening, at least on a yearly basis. Note that the typical size of your excursions in your graph are about 1 W/m^2. If we were really able to measure all of the radiative quantities to this degree of accuracy, then Trenberth wouldn’t be complaining (and people wouldn’t be using, at minimum, several years of ocean heat content data to diagnose how far the earth is out of radiative equilibrium).

I have gotten as far as “I think that I can show by observational datasets that the current paradigm is incorrect”. While I appreciate your invitation to look beyond that, the first step is establishing that the paradigm is in fact incorrect. I’m not doing anything past simple falsification right now.

I think that you conclusions are based on a combination of misinterpreting what the paradigm is saying and taking data beyond its accuracy. What you have done is confirmed what Trenberth said, which is that we are not currently able to measure all of the quantities to the sufficient fraction-of-a-Watt-per-meter-squared precision that would be needed to understand what is happening to energy in the climate system that is accumulating due to the small imbalances due to radiative forcing.

Pamela Gray
December 27, 2010 9:32 am

Let’s just say Stephen, that there is more than one change in stratospheric temperature to consider. Your thoughts on ozone depletion and signs of recovery in light of your thesis?
http://www.jstage.jst.go.jp/article/sola/5/0/53/_pdf

December 27, 2010 11:25 am

Very interesting post, thanks Willis.
If the Earth’s climate did not have means of self-correction, it would have gotten stuck in one of the extremes many, many centuries ago.
Happy New Year!

Stephen Wilde
December 27, 2010 11:53 am

Pamela,
That link is one which I have referred to regularly and it is fully accommodated in my proposals already.
May I suggest that you actually read what I say ?
Joel Shore said:
“Well, don’t be surprised to find yourself pretty lonely then in the scientific community, given that Miskolczi uses a data set that nobody seems to believe is accurate for the purposes that he uses it for.”
Since the scientific community has a serious problem explaining why AGW now seems so weak in the face of natural variability there isn’t much point joining them just to avoid loneliness. Anyway loneliness will disappear with a bit more evidence of what the natural processes are and can do.
harrywr2 said:
“Of course since we don’t know why the Hadley cells are acting the way they are it’s hard to simulate them.”
Now you are getting to the heart of it. They don’t know why ANY of the major circulation systems behave as they do.
My proposals provide just such an explanation and as time passes the real world is behaving as it should by my account.
If one adds a top down solar effect on the vertical temperature profile in the atmosphere and modulates that with bottom up oceanic variability then the pressure distribution changes and in particular mid latitude jetstream variations become fully explicable.
To achieve that top down solar effect I increasingly think that the secret is to regard radiative physics as largely irrelevant. What seems to matter most is chemical reactions caused by variations in the the energy and particles reaching us from the sun. Those reactions alter ozone quantities differentially at different levels to alter the vertical temperature profile so as to ultimately alter the height of the tropopause so as to alter the pressure distribution beneath.
The role of radiative physics is then merely to try and restore equilibrium after the chemical reactions have done their work.

Pamela Gray
December 27, 2010 4:01 pm

Interesting paper here on stratospheric temperature change. Three models were run, one with ozone depleting substances, one with greenhouse gases, and one with natural forcings. Natural forcings were quite capable of producing changes in ozone that then produced changes in the stratospheric temperature.
http://www.atmos-chem-phys-discuss.net/10/17341/2010/acpd-10-17341-2010.pdf

Pamela Gray
December 27, 2010 4:12 pm

Here’s another one that uses models and observations to describe the ozone-stratospheric temperature connection. It seems that several new and improved models are including a natural forcings only version.
hmmmmm.
http://www.springerlink.com/content/n56tw2k9xj546247/

Jim D
December 27, 2010 6:28 pm

I am still waiting for a reply to my previous comment.
http://wattsupwiththat.com/2010/12/23/some-of-the-missing-energy/#comment-559429
Why is Miskolczi ignoring the emitted longwave from the atmosphere when defining his optical depth? This is probably why he didn’t see any sensitivity to CO2 changes, and really explains a lot. To be relevant to climate you have to use the optical depth of the complete IR spectrum, which he didn’t. He just looked where you don’t expect to see a change, and indeed reported no change there. Is this the way to interpret his results?

Stephen Wilde
December 27, 2010 9:01 pm

Jim,
I don’t know the answer. I don’t even know whether your question is soundly based.
Pamela,
Very interesting and curious. I await developments.

Spector
December 28, 2010 2:07 pm

I think it is rather interesting that the top of the troposphere is literally the point where the atmosphere runs out of ‘steam’ or ‘water vapor’ as a result of condensation.
Whenever two water molecules join to form an H2O–H2O pair, the pair will initially have as much energy as if it were about 100 degrees C warmer then the individual molecules did before. I suspect that photon radiation from energetic condensing water vapor plumes may be an overlooked heat export mechanism. Note that an H2O–H2O pair actually has less mass than one single CO2 molecule.

David Socrates
January 13, 2011 11:20 am

Pamela Gray says:
December 25, 2010 at 5:40 am
I also find Stephen Wildes blog responses rather confusing but still gid value. If you are going to haul him over the coals for not being a good enough professional scientist to distinguish a hypothesis from a theory then I shal haul you over the coals for constantly confusing the words “affect” and “effect”.
So there!

Brian H
January 13, 2011 5:22 pm

DS;
A very effective rebuke! I can vicariously feel the sting from here. 😉

Stephen Wilde
January 13, 2011 7:12 pm

Thanks for the support, chaps but I’ve had to put up with worse than what Pamela said and she did back off later in the thread when she seemed to start seeing my point.
Equations and words are just different means of expression. Words are best for a general overview with equations best for detailed application.
In climate issues we don’t have the general overview right yet so in my opinion words are more helpful to a lay reader and more effective to get across the general concepts than equations.
Intellectual snobbery is ill advised since those who are best with equations are often the worst with words and vice versa.